CSIR NET BIOLOGY (September - 2022 Sift 1)
Previous Year Question Paper with Solution.

21. In crystalline NaCl, how many chloride ions surround each sodium ion?

(a) 4

(b) 6

(c) 8

(d) 10

Ans. (b)

Sol. In a crystalline NaCI, each sodium ion is surrounded by six chloride ions, and each chloride ion is surrounded by six sodium ions.

22. At which one of the following electron transport chain complexes does Antimycin A typically inhibit the respiratory chain?

(a) Complex I

(b) Complex II

(c) Complex III

(d) Complex IV

Ans. (c)

Sol. Antimycin A, a naturally occurring antibiotic, acts by binding to the cytochrome b of the enzyme complex III (also known as the cytochrome bcl complex).

23. Identify the ribose conformation in the nucleotide shown below.

(a) C2'-endo

(b) C2'-exo

(c) C3'-endo

(d) C5'-exo

Ans. (a)

Sol. Although the figure shows C2'-endo, but the given nucleotide contains ribose sugar and ribose sugar present in RNA. In this case, the C3'-endo conformation is characterized by the C31 carbon atom pointing inward towards the base. The C3'-endo conformation is the most common sugar pucker in RNA.

24. Which one of the following correctly describes the effect of a mutation in phosphofructokinase (PFK), that leads only to the loss of allosteric regulation by ATP?

(a) Decrease in the activity of PFK

(b) Increase in the activity of PFK

(c) Decrease in the amount of ADP generated by PFK

(d) Increase in the amount of ATP generated by PFK

Ans. (b)

Sol. ATP is an allosteric inhibitor of PFK, meaning that its binding to PFK reduces the enzyme's activity. Therefore, a mutation in PFK that specifically results in the loss of allosteric regulation by ATP will increase the activity of PFK.

25. Following statements are made regarding glycogen phosphorylase and glycogen synthase activities in relation to their phosphorylation status:

A. Phosphorylation of glycogen phosphorylase increases its activity

B. Phosphorylation of glycogen phosphorylase decreases its activity

C. Phosphorylation of glycogen synthase increases its activity

D. Phosphorylation of glycogen synthase decreases its activity

Which one of the following is a combination of correct statements?

(a) A and C

(b) B and C

(c) B and D

(d) A and D

Ans. (d)

Sol. A. Phosphorylation of glycogen phosphorylase increases its activity. This statement is correct. Phosphorylation of glycogen phosphorylase activates the enzyme, leading to an increase in its activity. Active glycogen phosphorylase catalyzes the breakdown of glycogen into glucose-1-phosphate, which is an essential step in glycogenolysis (glycogen breakdown).

D. Phosphorylation of glycogen synthase decreases its activity. This statement is also correct. Phosphorylation of glycogen synthase inactivates the enzyme, leading to a decrease in its activity. Inactive glycogen synthase cannot add glucose molecules to glycogen chains, resulting in the inhibition of glycogen synthesis.

26. The following statements were made to describe a typical collagen structure.

A. Collagen has a triple-helical domain structure which consists of three distinct á-chains.

B. The collagen triple helix is stabilized by isoprenyl bonds.

C. Each -chain has a left-handed polyproline II-type helix.

D. Each -chain is composed of multiple triplet sequences of Gly-Y-Z in which Y is commonly proline and Z is usually hydroxyproline.

Which one of the following options has all correct statements?

(a) A, C and D

(b) A, B and C

(c) A and B only

(d) B and D only

Ans. (a)

Sol. A. Collagen has a triple-helical domain structure which consists of three distinct a-chains. C. Each a-chain has a left-handed polyproline Il-type helix. D. Each a-chain is composed of multiple triplet sequences of Gly-Y-Z in which Y is commonly proline and Z is usually hydroxyproline.

27. The figure below represents the denaturation renaturation profile of a double stranded DNA in citrae buffer.

The percent of DNA that remains denatured at 30°C after cooling from 100°C is :

(a) < 25%

(b) 30-35%

(c) > 75%

(d) 65-70%

Ans. (a)

Sol. the percent of DNA that remains denatured at 30°C after cooling from 100°C is less than 25%. This indicates that a significant portion of the DNA has renatured (re-formed its double-stranded structure) at the lower temperature, leaving less than 25% of the DNA in the denatured single-stranded form..

28. The hemagglutinin protein in influenza virus contains a long á-helix, with 53 residues. Which of the following correctly describes the attributes of this α helix?

(a) The length is 75.6 Å, 14 turns, total of 102 Hydrogen bonds

(b) The length is 106 Å, 14 turns, total of 106 Hydrogen bonds

(c) The length is 75.6 Å, 14 turns, total of 104 Hydrogen bonds

(d) The length is 75.6 Å, 10 turns, total of 102 Hydrogen bonds

Ans. (a)

Sol. Length: 75.6 A (Angstroms)

Number of turns: 14 turns

Total number of hydrogen bonds: 102 hydrogen bonds

The length of the ƒ¿-helix is 75.6 A, it forms 14 turns, and it is stabilized by a total of 102 hydrogen bonds

29. Glycolysis and citric acid cycle contribute precursors to many biosynthetic pathways in plants. Column X lists names of the precursors and Column Y lists the product synthesized.

Which one of the following options represents the correct match between column X and Y?

(a) A-ii, B-iii, C-i and D-iv

(b) A-iii, B-ii, C-iv and D-i

(c) A-iv, B-i, C-iii and D-ii

(d) A-i, B-ii, C-iii and D-iv

Ans. (b)

Sol. Cellulose is a complex carbohydrate made up of repeating units of glucose (a hexose) One of the conversion pathways involves transamination, where pyruvate is converted into alanine. So, pyruvate can indeed be converted into alanine.

The pentose phosphate pathway generates ribose-5-phosphate, which is a precursor for the synthesis of nucleotides, including both RNA and DNA.

Oxaloacetate is an intermediate in the citric acid cycle (Krebs cycle) and can be converted into various other compounds

30. The figure below depicts a hypothetical scheme for synthesizing a target product in plants. (A), (B) and (C) are the precursors of a target product (D), whereas (E) is a by-product. The key enzymes of the pathway are indicated as E1-E6. To enhance the levels of target product (D), following strategies were tested:

A. Enhancing the activity of the enzyme E5 by over-expression and/or protein engineering.

B. Enhancing the activity of the enzyme E4 by over-expression and/or protein engineering.

C. Enhancing the levels of (C).

D. Blocking the activity of E6 by RNA-interference or CRISPR/Cas-mediated knockout.

Which of the above mentioned strategies are likely to provide the maximum enhancement of the target product compared to the by-product, if no feedback regulation exists for any of the enzymes in the pathway?

(a) A and B

(b) B and C

(c) C and D

(d) A and D

Ans. (d)

Sol. These two strategies (A and D) will work together to enhance the levels of the target product and minimize the production of the by-product, resulting in the maximum enhancement of the target product compared to the by-product in the absence of feedback regulation in the pathway.

31. A student was asked to plot a graph representing enzyme kinetic data for initial velocity, vo, and substrate concentration, [S] using any of the equations given below. The student used an equation for which neither X-axis nor Y-axis had independent variables. Which one of the following equations might the student have used?

(a) 1/vo = (Km/Vmax)1/[S] +1/Vmax

(b) [S]/vo = [S]/Vmax + (Km/Vmax)

(c) vo/[S] = (Vmax/Km) – vo / Km

(d) vo = Vmax [S]/Km + [S]

Ans. (c)

Sol. The equation that the student might have used, where neither the X-axis nor the Y-axis had independent variables, is:

Vo/[S] = (Vmax / Km) - Vo / Km.

32. Protein 'A' was subjected to different experiments:

(i) SDS-PAGE with/without -mercaptoethanol (β-ME) (ii) Fluorescence (iii) Far-UV and (iv) Near-UV CD spectra at pH 7.0 (blue) and 3.0 (red)

The results are shown below:

Which one of the following options provides the correct inference?

(a) Protein 'A' is an S-S bonded homotetramer and each subunit has a molecular mass of 50 kDa, folded at pH 7.0 and molten globule at pH 3.0.

(b) Protein 'A' has a molecular mass of 200 kDa, âME degrades the protein, low pH changes the conformation from helix to sheet.

(c) SDS denatures protein 'A' into different oligomeric states, low pH changes the conformation from helix to sheet.

(d) SDS promotes the formation of different oligomeric states of Protein 'A', low pH changes the conformation from sheet to helix.

Ans. (a)

Sol. The presence of S-S bonds in SDS-PAGE indicates that Protein 'A' contains covalent disulfide linkages between subunits. The homotetrameric nature suggests that it consists of four identical subunits.

33. The diffraction patterns of two forms of DNA (A and B) and statements related to these patterns are given below.

A. The diffraction pattern corresponding to A-DNA is shown by form 1 with 11.6 base pairs per turn.

B. The diffraction pattern of B-DNA is shown by form 2 with 11.6 base pairs per turn.

C. The diffraction pattern of A-DNA is shown by form 2 with 11.6 base pairs per turn.

D. The diffraction pattern of B-DNA is shown by form 1 with 10 base pairs per turn.

Which one of the following options have all correct statements?

(a) C and D

(b) A and B

(c) B only

(d) D only

Ans. (a)

Sol. C. The diffraction pattern of A-DNA is shown by form 2 with 11.6 base pairs per turn.

D. The diffraction pattern of B-DNA is shown by form 1 with 10 base pairs per turn.

34. A few organelles that are present in a eukaryotic cell are mentioned below:

A. Centrosomes

B. Peroxisomes

C. Nucleolus

D. Endosomes

Which one of the following options represents organelles that are not membrane-bound?

(a) A and B

(b) B and C

(c) A and C

(d) A and D

Ans. (c)

Sol. Centrosome and nucleolus are both non-membrane-bound organelles whereas, peroxisome and endosome are single membrane-bound organelles.

35. Individual chromosomes are clearly seen during which phase of the cell cycle?

(a) G0

(b) G1

(c) S

(d) M

Ans. (d)

Sol. The cell cycle has two main phases - interphase and M-phase. The period of actual division, corresponding to the visible mitosis, is called M phase (mitosis phase). During the M phase, the chromosomes condense and become visible under a light microscope.

36. Following statements were made about cell cycle regulation in eukaryotes:

A. Activity of maturation promoting factor (MPF) rises and falls in synchrony with the concentration of cyclin B.

B. Cdc25 phosphatase mediates removal of phosphate from the inhibitory tyrosine residue (Y15) to yield highly active MPF.

C. MPF specifically phosphorylates and depolymerizes lamin A and C, but not lamin B.

D. MPF phosphorylates H1 histone.

E. In Schizosaccharomyces pombe, overproduction of Wee1 protein decreases the length of G2 phase and extends the period of M phase by functioning as a mediator of MPF activity.

Which one of the following represents the combination of correct statements?

(a) A, B and C

(b) A, B and D

(c) B, C and E

(d) C, D and E

Ans. (b)

Sol. MPF is a complex of cyclin B and cyclin-dependent kinase 1 (CDK1), and its activity is tightly regulated by the levels of cyclin B, which fluctuate throughout the cell cycle. Cdc25 phosphatase is responsible for removing the inhibitory phosphate group from the tyrosine 15 (Y15) residue of CDK1, resulting in the activation of MPF.

MPF is involved in phosphorylating various target proteins during the cell cycle, and one of its targets is the H1 histone, which plays a role in chromatin condensation during mitosis.

37. Column A lists enzymes involved in cell cycle and typical function in a particular cell cycle phase is listed in Column B.

Which one of the following options represents the correct match between column A and column B?

(a) A-(iv), B-(iii), C-(ii), D-(i)

(b) A-(iii), B-(iv), C-(ii), D-(i)

(c) A-(i), B-(ii), C-(iii), D-(iv)

(d) A-(ii), B-(i), C-(iv), D-(iii)

Ans. (b)

Sol. Cdk1 (Cyclin-dependent kinase 1) is a key regulator involved in promoting the entry of cells into mitosis (M-phase) from the G2 phase of the cell cycle.

APC/C (Anaphase-promoting complex/cyclosome) is an E3 ubiquitin ligase complex that regulates the progression through various stages of the cell cycle.

Cdk2 (Cyclin-dependent kinase 2) is involved in regulating the entry of cells into the S-phase of the cell cycle.

Aurora B is a protein kinase that plays a central role in regulating cytokinesis, which is the final stage of the cell cycle

38. The table below lists the characteristics of specific tumor types (Column A) and their names (Column B).

Which of the following options represents the correct match between Column A and Column B?

(a) A-iii; B-iv; C-i; D-ii

(b) A-ii; B-iv; C-vi; D-iii

(c) A-iii; B-v; C-vi; D-i

(d) A-iv; B-v; C-i; D-ii

Ans. (b)

Sol. A tumor that has arisen from endodermal tissue = Carcinoma Carcinoma is a type of cancer that originates from epithelial cells, which are derived from endodermal and ectodermal tissues. Carcinomas often form solid tumors and can occur in various organs such as the lung, breast, colon, and prostate.

Cancer cells arisen from hematopoietic stem cells that do not grow as solid tumor = Leukemia Leukemia is a type of cancer that originates from hematopoietic stem cells in the bone marrow. Unlike solid tumors, leukemia involves the abnormal proliferation of blood cells, such as white blood cells, red blood cells, and platelets, and does not form solid masses.

A permanent change in the genome of a cell that results in abnormal growth = Transformation Transformation refers to the process by which a normal cell undergoes a permanent change in its genome, leading to uncontrolled and abnormal growth, which is characteristic of cancer cells.

A tumor that has arisen from mesodermal connective tissue = Sarcoma Sarcoma is a type of cancer that originates from mesodermal connective tissue, such as bone, muscle, cartilage, or fat. Sarcomas are characterized by their development in soft tissues and bones and can be further classified into various subtypes based on the specific tissue of origin.

39. Following statements are made about some of the abnormally expressing proteins in human cancers:

A. Increased telomerase expression always contributes to increased cell death in cancer cells.

B. Overproduction of anti-apoptotic protein (Bcl2) can lead to inappropriate cell survival and is associated with chronic lymphoblastic leukemia (CLL).

C. The E5, E6 and E7 proteins encoded by human papilloma virus (HPV) are tumor suppressors.

D. Overexpression of cyclin D1 or loss of p16 and Rb can cause inappropriate, unregulated passage through the restriction point in late G1.

Which of the following options represents the combination of all correct statements?

(a) A and B

(b) A and C

(c) B and D

(d) C and D

Ans. (c)

Sol. B. Overproduction of anti-apoptotic protein (Bcl2) can lead to inappropriate cell survival and is associated with chronic lymphoblastic leukemia (CLL).

D. Overexpression of cyclin D1 or loss of pl6 and Rb can cause inappropriate, unregulated passage through the restriction point in late G1

40. Which one of the following statements about the recognition of tRNAs by their cognate aminoacyl-tRNA synthetases is correct?

(a) Aminoacyl-tRNA synthetases recognize their cognate tRNAs by the exclusive recognition of their anticodons.

(b) Aminoacyl-tRNA synthetases recognize their cognate tRNAs by recognition of their anticodons in some tRNAs only.

(c) Aminoacyl-tRNA synthetases cannot aminoacylate a tRNA that lacks the conserved modifications in the loop.

(d) Aminoacyl-tRNA synthetases cannot aminoacylate a tRNA that lacks the conserved modifications in the DHU loop.

Ans. (b)

Sol. Attachment of an amino acid to tRNA involves a covalent linkage between the carboxyl group of the amino acid and the 2' or 3'-hydroxyl group of the adenine containing nucleotide at the 3' end of the tRNA. This process (called aminoacylation) is catalyzed by an enzyme called aminoacyl-tRNA synthetase (also known as aminoacyl-tRNA ligase). In general, aminoacyl-tRNA synthetases recognize the anticodon loops and acceptor arms of tRNA molecules.

41. Histone variants play important roles in chromatin function in mammalian cells. Which one of the following statements is correct in the context of the histone variants?

(a) Histone variants have been reported for H3 and H4 but not for H2A and H2B

(b) Histone variants have been reported for H3, H4, H2A but not for H2B

(c) Histone variants have been reported for H3, H4, H2B but not for H2A

(d) Histone variants have been reported for H3, H4, H2A and H2B

Ans. (d)

Sol. Histone variants have been reported for H3 (H3.1, H3.2, H3.3, H3.4, CENP-A), H4 (H4.1, H4.2, H4.3, H4.4), H2A (H2A.X, H2A.Z) and H2B (H2B.1, H2B.2, H2B.3).

42. Which one of the following statements made about the bacterial replisome is incorrect?

(a) The rate of forward movement of DnaB helicase along the template DNA increases.

(b) 10-fold when DnaB and DNA Pol III interact, thus ensuring that the helicase does not move ahead rapidly without the polymerase.

(c) The transient interaction of the primase with the helicase allows activation of primase activity by 1000-fold, promoting RNA primer synthesis.

(d) The length of the Okazaki fragments is typically restricted to 1000-2000 nucleotides.

(e) The E. coli oriC carries repeats of two sequence motifs: repeats of a 9-mer that collectively form the site at which the origin first becomes single-stranded, and repeats of a 13-mer to which the DnaA initiator protein binds.

Ans. (d)

Sol. The 9-mer repeats bind with DnaA protein. Binding of DnaA to the 9-mer repeats facilitates the unwinding of the DNA strands at 13-mer Region.

43. Which of the options correctly matches the proteins involved in transcription (Column A) with the DNA binding domains they carry (Column B)?

Column A Column B

A. TFIIIA i. Helix-turn helix

B. MyoD ii. Zinc finger

C. Jun iii. Helix loop helix

D. Cro iv. Leucine zipper

(a) A-iv, B-iii, C-i, D-ii

(b) A-ii, B-i, C-iv, D-iii

(c) A-iii, B-i, C-ii, D-iv

(d) A-ii, B-iii, C-iv, D-i

Ans. (d)

Sol. TFIIIA, MyoD and Jun are all transcription factors that bind to DNA as DNA binding proteins and regulate the expression of genes. Cro protein is a repressor protein that is encoded by the bacteriophage lambda cro gene. It is a DNA binding protein with helix turn helix motif.

44. The pathway for de novo biosynthesis of purine nucleotides involves the production of inosine monophosphate (IMP) that serves as a precursor for AMP and GMP synthesis. IMP has the base hypoxanthine whose structure is given below:

If hypoxanthine were incorporated into double stranded DNA, which of the following options correctly represents the order of its pairing preference?

(a) adenine > thymine > guanine > cytosine

(b) cytosine > adenine > thymine > guanine

(c) guanine > adenine > thymine > cytosine

(d) cytosine > thymine > adenine > guanine

Ans. (b)

Sol. This is because hypoxanthine can form base pairs with cytosine through three hydrogen bonds, with adenine through two hydrogen bonds, with thymine through one hydrogen bond, and with guanine through no hydrogen bonds.

45. Following statements were made about chromatin remodeling in eukaryotes:

A. Chromatin remodeling completely alters and/or slides the nucleosome, but cannot displace it.

B. Chromatin remodeling is an energy driven, developmentally regulated active process.

C. Histone acetylation is a reversible process, in which each direction of the reaction is catalyzed by different enzymes.

D. In general, acetylation of core histones reduces their affinity for DNA and destabilizes the chromatin structure, causing transcriptional repression.

E. Phosphorylation of Ser1 of histone H2A has been associated with transcription repression.

Which one of the following represents the combination of correct statements?

(a) A, B and C

(b) A, C and D

(c) B, C and E

(d) C, D and E

Ans. (c)

Sol. B. Chromatin remodeling is an energy-driven, developmentally regulated active process.

C. Histone acetylation is a reversible process, in which each direction of the reaction is catalyzed by different enzymes.

E. Phosphorylation of Serine of histone H2A has been associated with transcription repression

46. Following statements were made about some of the characteristics of the human genome:

A. Evidence derived by chromosome conformation capture (3C) suggests that each chromosome comprises a series of topologically associated domains.

B. Insulators typically mark the boundaries of topologically associated domains, preventing the genes within a domain from being influenced by the regulatory modules of an adjacent domain.

C. Presence of insulators does not overcome the positional effect after integration of a transgene into the genome.

D. Insulators can provide barrier against the spread of heterochromatin.

E. Insulator sequences are absent in the Drosophila genome, which suggests their essentiality in achieving highest degree of gene regulation in humans.

Which one of the following represents the correct combination of above statements?

(a) A, B and C

(b) A, B and D

(c) B, C and D

(d) C, D and E

Ans. (b)

Sol. A. Chromosome conformation capture (3C) techniques have revealed that the human genome is organized into topologically associated domains (TADs), which are regions of the chromosome that interact with each other more frequently than with regions outside the TAD.

B. Insulators are DNA sequences that act as boundary elements and help to define the boundaries of topologically associated domains. They prevent enhancers and other regulatory elements in one domain from interacting with genes in an adjacent domain.

D. Insulators can indeed serve as a barrier against the spread of heterochromatin, helping to maintain the distinct chromatin states of neighboring genomic regions

47. In eukaryotes, DNA replication must occur with extreme accuracy and only once to prevent the damaging effects of gene amplification. The following statements were made regarding possible mechanisms involved in achieving tight regulation of DNA replication:

A. High APC/C activity in mitosis and early G1 phase of the cell cycle that triggers the destruction of Cdt1 inhibitor geminin, thus allowing Cdt1 to be active in early G1 to load helicases.

B. Activation of S-Cdks that regulate the phosphorylation of specific initiator proteins in the S-phase.

C. MCM helicase loads at the S-phase of the cell cycle, so that replication begins only at S-phase.

D. Cdc6 and Cdt1 bind to the origin recognition complex (ORC) and help in pre-replicative complex assembly only after mitosis.

Which one of the options has all correct statements?

(a) A, B, and C only

(b) B, C, and D only

(c) A, B, and D only

(d) A, B, C, and D

Ans. (c)

Sol. A. High APC/C activity in mitosis and early G1 phase of the cell cycle that triggers the destruction of Cdt1 inhibitor geminin, thus allowing Cdt1 to be active in early G1 to load helicases.

B. Activation of S-Cdks that regulate the phosphorylation of specific initiator proteins in the S-phase.

D. Cdc6 and Cdt1 bind to the origin recognition complex (ORC) and help in pre-replicative complex assembly only after G1-S-Cdks are activated.

48. Excision repair systems replace a short stretch of DNA around the site of damage. The following statements are made about nucleotide excision repair in E. coli:

A. UvrB homodimer creates the nicks on one strand on both side of the lesion.

B. The 50-60 residue-long stretch of DNA between the two nicks is removed by the action of UvrD.

C. The gap generated is filled in typically by DNA polymerase I.

D. The distortion caused by the lesion is recognized and bound by UvrA-UvrB complex.

Which one of the following options represents the combination of all correct statements?

(a) A and B only

(b) A, B and D

(c) C and D only

(d) B, C and D

Ans. (d)

Sol. C. The gap generated is filled in typically by DNA polymerase I: After the damaged DNA segment is removed, the gap is typically filled in by DNA polymerase I in E. coli. DNA polymerase I has both 5' to 3' polymerase and 5' to 3' exonuclease activities, which allows it to replace the removed DNA segment.

D. The distortion caused by the lesion is recognized and bound by the UvrA-UvrB complex: In nucleotide excision repair, the UvrA-UvrB complex recognizes and binds to the distortion caused by the DNA lesion. This complex initiates the repair process by recruiting UvrC, which makes the incisions on both sides of the lesion.

49. Although introns are not a part of the processed transcript that gets translated, they are important for several reasons. The following statements are made with reference to the possible ways in which introns are crucial to cell survival.

A. They permit the generation of different protein products from the same gene.

B. They may encode miRNAs which modulate the expression of genes.

C. They often encode peptides which play a role in regulating gene expression.

D. They promote export of certain mRNAs through the recruitment of transport proteins by the Exon Junction Complex (EJC).

E. They play a role in mRNA surveillance through the modulation of nonsense-mediated mRNA decay via the Exon Junction Complex (EJC).

Which one of the following options represents the combination of all correct statements?

(a) A, B and E only

(b) A, C and D only

(c) A, B, D and E

(d) B, C and D only

Ans. (c)

Sol. Alternative splicing of introns allows a single gene to produce multiple mRNA isoforms, which can lead to the synthesis of different protein products with varying functions.

Some introns can contain sequences that serve as precursors for microRNAs (miRNAs).

The Exon Junction Complex (EJC) forms at the junctions between exons and introns during splicing Intron-containing mRNAs are more likely to be subjected to NMD when splicing is disrupted, contributing to mRNA quality control.

50. The initiation of transcription is a complex process involving promoter recognition, conversion of the initiation complex from closed to open form, abortive initiation events, and finally promoter escape. The following statements are made regarding these steps in transcription initiation:

A. Promoter escape in bacteria is usually accompanied by the release of the sigma factor from the RNA polymerase holoenzyme complex.

B. Abortive initiation events in prokaryotes result in the formation of short transcripts ~10 nucleotides in length while such events in eukaryotes result in formation of transcripts ~75 nucleotides in length.

C. Promoter escape in eukaryotes is accompanied by the phosphorylation of the RNA polymerase large subunit on its C-terminal domain (CTD).

D. Promoter recognition in bacteria is governed by the sigma factor which binds to the -10 and -35 regions of the promoter followed by recruitment of the RNA Pol II core enzyme to form the holoenzyme.

Which one of the following options represents the combination of all correct statements?

(a) A and C only

(b) B and D only

(c) A, C and D

(d) A and D only

Ans. (a)

Sol. In bacterial transcription initiation, the sigma factor is involved in promoter recognition and binding to specific sequences (-10 and -35 regions) in the promoter. After the RNA polymerase holoenzyme initiates transcription and moves away from the promoter, the sigma factor is released, leaving the core RNA polymerase to continue elongating the transcript.

In eukaryotic transcription initiation, RNA polymerase II (Pol II) undergoes phosphorylation of its C-terminal domain (CTD) during the transition from initiation to elongation phase. This phosphorylation is associated with promoter escape, allowing the RNA polymerase to move away from the promoter and start elongating the transcript.

51. Fidelity of protein synthesis depends to a large extent on the accuracy of aminoacylation of tRNAs with correct amino acids. However, given that the side chains of many amino acids are not sufficiently different, aminoacyl-tRNA synthetases (aaRS) are often prone to misacylate the tRNAs. One such example of misacylation is of tRNAThr by ThrRS. In this context, following statements are being made about E. coli ThrRS.

A. It misacylates tRNAThr equally with Ser and Cys

B. It possesses a distinct editing site that preferentially deacylates the misacylated tRNAThr

C. The editing of the misacylated tRNAThr occurs frequently in cis before the release of the misacylated tRNAThr

D. It possesses a distinct editing site that does not discriminate between the misacylated tRNAThr and Thr-tRNAThr

E. The aminoacylation and the editing sites of ThrRS are the same.

Choose the option that represents all correct statements.

(a) A and B

(b) B and C

(c) C and D

(d) D and E

Ans. (b)

Sol. B. It possesses a distinct editing site that preferentially deacylates the misacylated tRNAThr

C. The editing of the misacylated tRNAThr occurs frequently in cis before the release of the misacylated tRNAThr.

52. A 'nonsense' mutation in the protein coding region of an upstream gene of a group of genes in an operon often leads to depletion of the downstream gene products. This is a classic example of the phenomenon of "polar effect" of the mutation. Following statements are being made about this phenomenon.

A. It occurs primarily because the termination codon generated in the upstream open reading frame (ORF) leads to termination of protein synthesis depleting the ribosomes for translation of the downstream ORFs but it does not affect the process of transcription.

B. The phenomenon of polar effects of mutation occurs only in the operons where the point mutation leading to creation of 'nonsense' mutation also leads to formation of a stem-loop structure resulting in Rho-independent termination.

C. While the presence of termination codon in the upstream ORF may deplete the ribosomes that travel down to the downstream ORF, the depletion of ribosomes downstream of the 'nonsense' mutation allows loading of the Rho factor that then results in premature transcription termination.

D. Presence of the suppressor tRNA reading the 'nonsense' codon generated by the mutation, is essential for causing a polar effect of the mutation.

E. Presence of the suppressor tRNA reading the 'nonsense' codon generated by the mutation diminishes the consequences of the polar effects.

Choose the option that represents all correct statements.

(a) A and C

(b) B and D

(c) C and E

(d) A and D

Ans. (c)

Sol. The presence of a termination codon in the upstream ORF can lead to the termination of translation, causing ribosomes to disengage prematurely and depleting ribosomes downstream. If a suppressor tRNA is present that can read the nonsense codon generated by the mutation, it can suppress the termination of translation at the nonsense codon, leading to the continuation of protein synthesis downstream

53. Regulation of mRNA translation is a major mechanism that maintains stoichiometric availability of ribosomal proteins (r-proteins) to rRNA molecules they bind to. Translational regulation is facilitated by general occurrence of the r-protein genes in several operons containing multiple genes. Which one of the following represents an established mechanism to ensure optimal production of the r-proteins in E. coli, when the r-proteins accumulate in free form (molar excess over rRNA)?

(a) The free r-protein(s) often bind to corresponding DNA sequence and activate transcription of rRNA genes to increase rRNA availability.

(b) The free r-protein(s) bind to RNA polymerase and represses transcription of the r-protein genes to decrease the availability of their mRNAs.

(c) The free r-protein(s) bind to the mRNA(s) and downregulate their translation.

(d) The free r-protein(s) bind free NTPs which then activates their cryptic ribonuclease activity leading to the degradation of their mRNAs.

Ans. (c)

Sol. When the levels of r-proteins exceed the demand for ribosome assembly, the excess free r-proteins can bind to specific regions on their own mRNA molecules. This binding inhibits translation initiation, leading to a downregulation of the translation of r-protein genes.

54. The following represents sequences of different alleles of a gene found in a family represented by mother (allele1/ allele2), father (allele1/allele 2) and their two sons: Son1 (allele1/allele 2) and Son 2 (allele1/allele2). Further, a new mutation was observed in one of the alleles of the son, which is marked with a triangle.

The following statements were made about the mutation:

A. The mutation arose in the germline of the father.

B. The mutation arose in the son.

C. The given DNA sequences are present on the X chromosome.

D. There is a possibility to use RFLP for tracking this variation.

Which one of the following options presents a combination of correct statements?

(a) B only

(b) A and D

(c) A and C

(d) B and D

Ans. (b)

Sol. A. The mutation arose in the germline of the father. This statement is correct. The mutation is observed in one of the alleles of the son (marked with a triangle), and it is present in both the father's alleles. Since the sons inherit one allele from each parent, the mutation must have arisen in the germline of the father and passed on to the sons.

D. There is a possibility to use RFLP for tracking this variation. This statement is also correct. RFLP (Restriction Fragment Length Polymorphism) analysis can be used to detect and track genetic variations in DNA sequences. In this case, the mutation can potentially create or remove a restriction site for a specific enzyme, leading to differences in the fragment lengths when the DNA is digested with that enzyme. By using RFLP analysis, researchers can identify and track the presence of the mutation in the family members' DNA samples.

55. The following table enlists different ways of carrying out reverse genetics (Column X) and different strategies to achieve the same (Column Y).

Which one of the following options is a correct match between Column X and Y?

(a) A – i and iv; B- iii; C- ii and v

(b) A – ii and iv; B- iii and v; C- i

(c) A- i and iv; B- iii and v; C- ii

(d) A- ii and iv; B- i and iii; C – v

Ans. (b)

Sol. A. Random mutagenesis = ii. Transposable elements

Random mutagenesis involves the introduction of random mutations into the genome, and transposable elements are genetic elements that can move to different locations within the genome, often leading to mutations.

B. Targeted mutagenesis = iii. Homologous recombination

Targeted mutagenesis involves the deliberate introduction of specific mutations in a defined region of the genome, often using homologous recombination to precisely replace or modify a particular gene or DNA sequence.

C. Phenocopying = i. RNAi

Phenocopying is a process where a specific phenotype is induced by experimental manipulation, and RNA interference (RNAi) is a technique used to suppress gene expression by introducing double-stranded RNA that specifically targets and degrades the mRNA of the gene of interest, leading to a phenocopy of the gene's loss-of-function phenotype.

56. Which one of the following combinations of CD molecules and their associated functions is matched incorrectly?

(a) CD1: Antigen-presenting proteins that present antigenic peptides to T-cell receptors on natural killer T cells (NKT)

(b) CD8: Thymic differentiation marker for T cells

(c) CD11a: A membrane glycoprotein that provides cell-cell adhesion by interaction with ICAM 1 (intercellular adhesion molecule 1)

(d) CD14: Activates innate immune responses by transferring LPS-LBP complex to TLR4.

Ans. (a)

Sol. Conventional T cells : Cytoxoic T cells : Peptide antigens presented by class I MHC

Helper T cells : Peptide antigens presented by class II MHC.

Natural killer T cells : Non-peptide antigens presented by class I MHC-related protein CD1.

57. Which one of the following is a small sulfated peptide that is secreted by a rice pathogenic bacterium, Xanthomonas oryzae to modulate motility, biofilm formation and virulence?

(a) Coronatine

(b) N-acylhomoserine lactones

(c) Ax21

(d) EPS

Ans. (c)

Sol. A × 21 is a small sulfated peptide that is secreted by the rice pathogenic bacterium Xanthomonas oryzae. It is a quorum sensing molecule, which means that it is released by the bacteria in response to a certain population density.

58. While studying pathogenic bacteria, a protein with the following features was identified:

A. It was secreted during infection conditions, but not in in-vitro cultures

B. It was also observed to be present in the membranous fraction in traces, which was released upon bacterial lysis

C. It had a heat labile N-terminal enzymatic domain that binds MHC molecules, stimulating T cells non-specifically

D. It had a C-terminal non-enzymatic domain which was highly antigenic and heat-stable

How will you best classify the toxic nature of this protein?

(a) An endotoxin

(b) Superantigen

(c) Pore-forming toxin

(d) A-B toxin

Ans. (b)

Sol. Superantigen is a class of microbial toxin or protein that binds to the variable region of the T-cell receptor (TCR) and major histocompatibility complex (MHC) class II molecules, resulting in the non-specific activation of a large number of T cells.

Pore-forming toxins insert itself into the host cell membrane and make an open channel (pore).

A-B toxins are intracellular toxins. The B-subunit attaches to target regions on cell membranes and the A-subunit enters through the membrane and possesses enzymatic activity.

59. Lr34, a broad-spectrum disease resistance gene in wheat, encodes for a putative:

(a) Serine hydroxymethyl transferase

(b) ABC transporter

(c) Host-specific toxin

(d) TIR-NB-LRR protein

Ans. (b)

Sol. Lr34 is a broad-spectrum disease resistance gene in wheat that encodes for an ABC transporter.

60. If you want to selectively kill the newly dividing mammalian cells in a cell culture assay, which of the following methods will you use?

(a) Exposure to UV radiation at 250 nm.

(b) Treatment with 5-ethynyl-2-deoxyuridine (EdU), followed by doxorubicin hydrochloride treatment.

(c) Treatment with 5-bromo-2-deoxyuridine (BrdU), followed by UV-A exposure

(d) Tritiated thymidine treatment followed by vinblastine treatment

Ans. (c)

Sol. 5-bromo-2-deoxyuridine (BrdU) is a thymidine analog that is incorporated into DNA during the S phase of the cell cycle. When BrdU-labeled cells are exposed to UV-A light, the BrdU is converted to a photoreactive compound that damages the DNA, leading to cell death. This method is selective for newly dividing cells because only cells that are actively incorporating BrdU into their DNA will be killed

61. The signal transduction pathway involved in glycogen metabolism triggered in the liver by the hormone epinephrine involves the following steps:

A. Activation of G protein (Gαβγ) by the activated receptor

B. Protein kinase A activation

C. Second messenger generation (3',5' cyclic AMP)

D. Adenylyl cyclase activation

Which one of the following combinations describes these processes in the right order?

(a) A-D-C-B

(b) A-C-D-B

(c) D-A-B-C

(d) A-D-B-C

Ans. (a)

Sol. A. Activation of G protein (Gapy) by the activated receptor

D. Adenylyl cyclase activation

C. Second messenger generation (3',5' cyclic AMP)

B. Protein kinase A activation

62. The immune system of thymectomized and lethally irradiated (A X B) F1 mice (H-2a/b) were reconstituted by grafting thymus from Strain B mice (H-2b) and with (A X B) F1 bone marrow cells. These mice were then infected with lymphocytic choriomeningitis virus (LCMV). The CD8+ cytotoxic T lymphocytes from the reconstituted mice were then co-cultured with the following cells:

A. Uninfected fibroblast cells from (A X B) F1 mouse

B. Uninfected fibroblast cells from strain-A mouse

C. Uninfected fibroblast cells from strain-B mouse

D. LCMV infected fibroblast cells from (A X B) F1 mouse

E. LCMV infected fibroblast cells from strain-A mouse

F. LCMV infected fibroblast cells from strain-B mouse

Which of the options below indicate the cells that will be lysed by the CD8+ cytotoxic T lymphocytes from the reconstituted mice?

(a) A, C, D and F only

(b) D, E and F only

(c) D and F only

(d) B and F only

Ans. (c)

Sol. These are the cells that will be targeted and lysed by the CD8+ cytotoxic T lymphocytes from the reconstituted mice. LCMV-infected fibroblast cells from both the (A X B) Fi mouse and strain-B mouse will be recognized and killed by the CD8+ T cells.

63. There are many superfamilies of adhesion proteins, which play a central role in cell-cell adhesion in animals. Ig superfamily proteins are one such adhesion proteins. Which one of the following statements about Ig superfamily proteins is incorrect?

(a) The white blood cell proteins recognized by endothelial cell integrins are called ICAM (intercellular cell adhesion molecule) or VCAM (vascular cell adhesion molecules)

(b) These are called Ig superfamily because they contain one or more extracellular Ig-like domains that are characteristic of antibody molecules.

(c) ICAM and VCAM mediate heterotrophic binding to integrin, whereas NCAM (neural cell adhesion molecule) mediates homotrophic binding.

(d) They contain large quantities of sialic acid which inhibit adhesion by charge-based repulsion contributing to fine tuning of cell-cell adhesion.

Ans. (d)

Sol. The white blood cell proteins recognized by endothelial cell integrins are indeed called ICAM (intercellular cell adhesion molecule) or VCAM (vascular cell adhesion molecules)

64. Interferons and are cytokines produced and secreted by animal cells after infection by viruses. Which one of the following statements about interferons is incorrect?

(a) When mammalian cells are incubated with different interferons, activation of STATs (Signal Transducers and Activators of Transcription) links stimulation of cell surface receptors with gene expression.

(b) Increase in the expression of genes after addition of IFNα results only after activation of the interferon-stimulated response element (ISRE).

(c) The receptors of all three interferons belong to the Ig superfamily of receptors and do not lead to downstream phosphorylation events.

(d) An important feature of interferon-STAT signaling pathway is its specificity: each type of interferon induces transcription of a unique subset of genes.

Ans. (c)

Sol. the receptors of interferons and do not belong to the Ig superfamily of receptors, and they do lead to downstream phosphorylation events upon binding to their respective interferons. Interferon receptors are typically associated with Janus kinases (JAKs) and signal transducers and activators of transcription (STATs), leading to the activation of specific genes involved in antiviral responses.

65. Mouse IgM (whole molecule) was injected into rabbit to generate antiserum. Which one of the following mouse antibody components (the same mouse from which IgM molecules were used for immunization) has the possibility to be recognized using the rabbit antiserum in Western blotting?

(a) IgG F(ab')2 fragment only

(b) J chain only

(c) IgG Fc fragment only

(d) Both IgG F(ab')2 fragment and J chain

Ans. (d)

Sol. The rabbit antiserum generated against mouse IgM (whole molecule) in this context can recognize both the IgG F(ab')2 fragment and the J chain of mouse IgM in Western blotting. This is because the rabbit antiserum is likely to contain antibodies against different epitopes of the mouse IgM, including the F(ab')2 fragment and the J chain, which are present in the intact IgM molecule. Thus, when used in Western blotting, the rabbit antiserum can detect these components of the mouse IgM antibody.

66. Which one of the following statements is incorrect?

(a) Transient rise in Ca2+ is necessary for egg activation in mammals.

(b) Sperm induces egg activation and does not involve Ca2+.

(c) In many organisms, eggs secrete diffusible molecules that attract and activate sperm.

(d) Capacitated mammalian sperm can penetrate the cumulus and bind the zona pellucida.

Ans. (b)

Sol. Fertilization generally consists of four major events:

1. Contact and recognition between sperm and egg.

2. Regulation of sperm entry into the egg.

3. Fusion of the genetic material of sperm and egg.

4. Activation of egg metabolism to start development - Calcium plays a crucial role in the activation of egg metabolism during fertilization and early development.

67. In which one of the following developmental events, the fate of maternal somatic cell is determined first, which then determines the fate of the developing embryo?

(a) The specification of primary organizer in amphibian embryo.

(b) The specification of dorso-ventral axis in Drosophila.

(c) The formation of the vulval precursor cells during development of C. elegans.

(d) Specification of the micromeres in case of sea urchin.

Ans. (b)

Sol. In Drosophila (fruit flies), the specification of the dorso-ventral axis is crucial for the proper development of the organism. The process of dorso-ventral axis formation begins during early embryonic development. Here are the key steps involved:

Egg Formation: Drosophila eggs are laid with an anterior-posterior (AP) polarity. The future ventral side of the embryo will face down, while the dorsal side will be oriented upwards. Gurken Localization: Within the oocyte, an mRNA molecule called gurken is localized to the posterior end of the future dorsal side. Gurken is important for the establishment of the dorso-ventral axis.

68. Which one of the following floral mutants shows the pattern 'sepals-petals-petals' repeated several times?

(a) agamous (ag)

(b) apetala1 (ap1)

(c) apetala3 (ap3)

(d) pistillata (pi)

Ans. (a)

Sol. Class A genes are responsible for specifying the identity of sepals and petals. The class A genes include APETALA1 (API) in Arabidopsis. Class B genes play a role in determining the identity of petals and stamens. Class B genes include APETALA3 (AP3) and PISTILLATA (PI) in Arabidopsis. Class C genes are involved in specifying the identity of stamens and carpels. The class C genes include AGAMOUS (AG) in Arabidopsis. The expression of class A genes and class C genes is postulated to be mutually antagonistic.

69. The table below lists cleavage pattern and names of species.

Cleavage pattern Species

A. Isolecithal bilateral i. Amphibians

B. Mesolecithal radial ii. Birds

C. Centrolecithal superficial iii. Tunicates

D. Telolecithal discoidal iv. Insects

Match the cleavage patterns with the species.

(a) A-i; B-ii; C-iii; D-iv

(b) A-ii; B-iv; C-i; D-iii

(c) A-iv; B-i; C-iii; D-ii

(d) A-iii; B-i; C-iv; D-ii

Ans. (d)

Sol.

70. The HIPPO signaling pathway is important for cell proliferation. It is regulated by the protein kinases MST1/2 and LATS1/2, and the transcriptional activators YAP and TAZ. Accordingly, the following events may be observed within a cell.

A. Activation of TEADs by phosphorylated YAP/TAZ.

B. Activation of YAP/TAZ on phosphorylation by MST/LATS

C. Activation of TEADs by dephosphorylated YAP/TAZ

D. Inactivation of MST / LATS by repressors

Which one of the following situations can support progression of cancer?

(a) A and B

(b) B and C

(c) C and D

(d) A and D

Ans. (c)

Sol. It is important to note that the HIPPO signaling pathway is a complex regulatory network involved in various cellular processes, including tissue homeostasis, organ size control, and cancer development. Dysregulation of this pathway, particularly with the factors mentioned in options C and D, can lead to cancerous growth.

71. In mice, the gene encoding Tbx5 is transcribed in limb fields of the forelimbs, while the genes encoding Islet1, Tbx4 and Pitx1 are expressed in presumptive hindlimbs.

Following statements are made about limb development in mouse:

A. Loss of Tbx5 gene results in complete failure of forelimb formation.

B. Hindlimb bud growth and initial patterning appears normal when Tbx4 is knocked out, although leg development is arrested prematurely.

C. Misexpression of Pitx1 in forelimb ceases development of muscles, bones and tendons.

D. When Islet1 is inactivated specifically in the lateral plate mesoderm, the hindlimbs still develop.

Which one of the following options represents a combination of correct statements?

(a) A and B

(b) A and C

(c) B and C

(d) C and D

Ans. (a)

Sol. A. Loss of Tbx5 gene results in complete failure of forelimb formation.

B. Hindlimb bud growth and initial patterning appears normal when Tbx4 is knocked out, although leg development is arrested prematurely.

72. The following statements are made about mammalian development:

A. Zygote is a totipotent stem cell.

B. The cells of inner cell mass are said to be pluripotent.

C. The three regulatory transcription factors, Oct4, Nanog and Sox2 help maintain pluripotency of the inner cell mass.

D. Cdx2 upregulates Oct4 and Nanog.

Which one of the following options represents the correct combination of the statements?

(a) A and B

(b) B and C

(c) C and D

(d) A and D

Ans. (b)

Sol. B. The cells of the inner cell mass are said to be pluripotent: The inner cell mass (ICM) of the blastocyst is composed of pluripotent cells, which have the ability to differentiate into all cell types of the three primary germ layers (endoderm, mesoderm, and ectoderm) but cannot give rise to the extraembryonic tissues like the placenta.

C. The three regulatory transcription factors, Oct4, Nanog, and Sox2 help maintain pluripotency of the inner cell mass: Oct4, Nanog, and Sox2 are important transcription factors that play a key role in maintaining pluripotency in embryonic stem cells, including the cells of the inner cell mass.

73. In the table below Column I lists terms related to development and Coumn II contains their description not in a sequential manner :

Select the option with all correct matches Column I and Column II.

(a) A-i, B-iii, C-ii, D-iv

(b) A-ii, B-i, C-iv, D-iii

(c) A-iii, B-i, C-ii, D-iv

(d) A-iv, B-ii, C-i, D-iii

Ans. (c)

Sol. Koller's sickle = Local thickening of the epiblast formed at the posterior edge of the area pellucida.

Primary hypoblast = The delaminated cells from epiblast forming islands.

Primitive groove = Homologous to the amphibian blastopore.

Hensen's node = Equivalent of the dorsal blastopore lip of the amphibian embryo.

74. C. elegans embryo uses both autonomous and conditional modes of specification. The following statements are about specification of cell lineages:

A. The signals from the P1 blastomere instructs the EMS cell for its further development.

B. In the absence of the instructive signal, the EMS cell will divide into two MS cells.

C. In the absence of the POP-1 signal, the prospective MS cell can take up the E fate.

D. The MOM-2 protein from the blastomere neighbouring the EMS cell instructs the dividing EMS blastomere to take up MS fate.

Which one of the following options represent both correct statements?

(a) A and B

(b) B and C

(c) C and D

(d) A and D

Ans. (b)

Sol. B. In the absence of the instructive signal, the EMS cell will divide into two MS cells. C. In the absence of the POP-1 signal, the prospective MS cell can take up the E fate.

75. The interactions that maintain polarity during Planaria regeneration is shown in the figure below:

Following statements regarding these interactions were made:

A. When Notum expression is knocked down, the anterior facing blastema will still form a head.

B. When Notum is expressed in the posterior end, Planaria with two heads will be formed.

C. When Wnt pathway is blocked, the resulting Planaria will have heads on both the ends.

D. High levels of Erk inhibit head specification.

Which one of the following options represents the correct combination of the statements?

(a) A and C

(b) B and C

(c) C and D

(d) A and D

Ans. (b)

Sol. Notum is a protein that antagonizes the Wnt signaling pathway, which is involved in establishing anterior-posterior polarity during Planaria regeneration.

When this pathway is blocked or inhibited, it disrupts the establishment of anterior-posterior polarity, leading to the formation of heads on both ends of the regenerated Planaria

76. Given below are the list of proteins (Column X) and their functions (Column Y) during floral induction.

Which one of the following options represents the correct match between column X and column Y?

(a) A - i, B - ii, C - iii, D – iv

(b) A - iii, B - iv, C - i, D - ii

(c) A - ii, B - i, C - iv, D – iii

(d) A - iv, B - iii, C - ii, D - i

Ans. (d)

Sol. A. FLOWERING LOCUS C (FLC) = iv. A strong repressor of flowering FLC is a gene that acts as a strong repressor of flowering in many plants. It delays the transition from vegetative growth to flowering.

B. FLOWERING LOCUS D (FD) = iii. Regulate target genes that mediate the reprogramming of meristem to produce flowers FD is a gene that regulates target genes involved in the reprogramming of the shoot apical meristem to produce flowers. It is a key component in the flowering pathway.

C. FLOWERING LOCUS T (FT) = ii. A mobile signal that induces flowering FT is a gene that encodes a mobile signaling molecule that moves from leaves to the shoot apical meristem. It induces flowering and plays a central role in the regulation of flowering time.

D. FRIGIDA (FRI) = i. An activator of FLC FRIGIDA (FRI) is a gene that acts as an activator of FLOWERING LOCUS C (FLC). It promotes the expression of FLC, leading to the repression of flowering.

77. In Drosophila, balancer chromosomes are used to keep all the alleles on one chromosome together. A balancer contains multiple inversions; so that when it recombines with the corresponding wild type chromosome, no viable cross over products are formed. Balancers also carry an allele for a dominant phenotype.

A Drosophila male with sepia eye color is crossed to a female carrying a third chromosome balancer (TM6B). The allele for sepia phenotype (se) is located on chromosome 3 and is recessive to the wild type eye color. The dominant marker for TM6B is a tubby phenotype. Further, an individual homozygous for TM6B balancer does not survive. F1 progeny with tubby phenotype is sib-mated.

The F2 progeny is expected to have:

(a) only sepia eye color

(b) sepia, tubby and wild type flies in a ratio of 1:2:1

(c) sepia and tubby flies in a ratio of 1:2

(d) sepia and wild type flies in ratio of 3:1

Ans. (c)

Sol. In the F1 generation, when a Drosophila male with sepia eye color (se/se) is crossed to a female carrying a third chromosome balancer (TM6B), the F1 offspring will be heterozygous for both sepia and tubby (se/TM6B).

78. Which of the following statements regarding chlorophyll is not correct?

(a) Chlorophyll-a has a –CH3 group in its porphyrin-like ring structure.

(b) Chlorophyll-b has –CHO group in its porphyrin-like ring structure.

(c) Only chlorophyll-a, but not chlorophyll-b, has a Mg++ coordinated at the centre of the porphyrin-like ring structure.

(d) The long hydrocarbon tails of chlorophyll anchors them in the photosynthetic membrane.

Ans. (c)

Sol.

79. Which of the following nitrogen containing compounds is formed during deamination of organic nitrogen in plants?

(a) NO

(b)

(c)

(d)

Ans. (d)

Sol. The product of deamination of organic nitrogen in plants is ammonia. Deamination is a process that removes an amino group from an organic molecule. The amino group is removed from the amino acid and converted to ammonia.

80. The product of nahG gene of Pseudomonas putida catalyzes the metabolism of salicylic acid to which one of the following compounds?

(a) Benzoic acid

(b) Methyl salicylate

(c) Catechol

(d) Benzoyl-CoA

Ans. (c)

Sol. The product of the nahG gene of Pseudomonas putida catalyzes the metabolism of salicylic acid. The nahG gene encodes the enzyme salicylate 1-monooxygenase (a flavin-dependent monooxygenase), which catalyzes the oxidation of salicylic acid to catechol.

81. The following are selected plant apomorphies:

A. Development of xylem

B. Development of cuticle

C. Development of independent sporophyte

D. Development of eustele

Which option represents the correct evolutionary sequence of the above?

(a) A-D-B-C

(b) C-A-B-D

(c) B-C-A-D

(d) C-B-D-A

Ans. (c)

Sol. The development of cuticle is considered one of the earliest apomorphies In the plant kingdom, providing a protective waxy layer on the surface of the plant to reduce water loss. The development of an independent sporophyte was next major evolutionary development that allowed plants to colonize land, the development of xylem, responsible for transporting water and minerals, can be considered as a subsequent apomorphy that evolved after the establishment of an independent sporophyte. The development of a eustele was a later evolutionary innovation that allowed plants to grow taller.

82. Young seedlings of Arabidopsis plants are exposed to the following light conditions:

A. Far-Red light followed by Red light

B. Far-Red light followed by Red light and then Dark phase

C. Red light followed by Far-Red light

D. Dark phase followed by Far-Red light and then Red light

E. Far-Red light followed by Dark phase and then Red light

F. Red light followed by Dark phase and then Far-Red light

Which of the above conditions will lead to photomorphogenesis?

(a) A, B and E

(b) B and F

(c) C and F

(d) A, D and E

Ans. (d)

Sol. A. Far-Red light followed by Red light

D. Dark phase followed by Far-Red light and then Red light

E. Far-Red light followed by Dark phase and then Red light

83. Following are certain statements regarding gibberellic acid (GA) signal transduction:

A. DELLA proteins negatively regulate GA signalling.

B. Degradation of GA receptor (GID1) is mediated by DELLA proteins.

C. Ubiquitination and subsequent degradation of DELLA proteins are independent of GID1.

D. GA binding to GID1 promotes binding of GID1 to DELLA proteins.

Which one of the following combination of statements is correct?

(a) A and B

(b) B and C

(c) C and D

(d) A and D

Ans. (d)

Sol. A. DELLA proteins negatively regulate GA signaling. D. GA binding to GID1 promotes binding of GID1 to DELLA proteins.

84. There are different kinds of reactive oxygen species (ROS) generated in plants. The following are the some of the statements related to ROS and its scavenging:

A. H2O2 is relatively more stable and travels relatively long distances.

B. ROS is scavenged only through enzymatic reactions.

C. Ascorbate-glutathione cycle is associated with the scavenging of ROS.

D. Monodehydroascorbate reductase is not an antioxidant enzyme.

Which one of the following combination of statements is correct?

(a) A and C

(b) A and D

(c) B and C

(d) B and D

Ans. (a)

Sol. A. H2O2 is relatively more stable and travels relatively long distances. This statement is correct. Hydrogen peroxide (H2O2) is a relatively stable reactive oxygen species (ROS) compared to other ROS, such as superoxide (O2-) and hydroxyl radicals (OH·). Due to its stability, H2O2 can travel relatively long distances within plant cells and tissues, leading to signaling and oxidative damage at distant sites.

C. Ascorbate-glutathione cycle is associated with the scavenging of ROS. This statement is correct. The ascorbate-glutathione cycle is an essential enzymatic pathway involved in the scavenging and detoxification of ROS in plants. It helps to neutralize ROS, including hydrogen peroxide (H2O2), and maintain cellular redox balance.

85. Following statements were made to explain the intracellular transport that occurs through apoplastic, symplastic and transcellular routes in plants:

A. Apoplastic transport mostly occurs through cell-wall.

B. Apoplastic transport mostly involves plasmodesmata.

C. Symplastic transport predominantly occurs through plasmodesmata.

D. Transcellular transport mostly occurs through tonoplast via vacuoles.

Which one of the following combination of statements is correct?

(a) A, B and D

(b) B, C and D

(c) A, C and D

(d) A, B and C

Ans. (c)

Sol. Apoplastic transport occurs through the cell wall.

Symplastic transport predominantly occurs through plasmodesmata.

Transcellular transport mostly occurs through the tonoplast via vacuoles.

86. Which one of the following small molecule neurotransmitters is not synthesized from tyrosine?

(a) Epinephrine

(b) Dopamine

(c) Serotonin

(d) Norepinephrine

Ans. (c)

Sol. Catecholamines (Dopamine, norepinephrine and epinephrine): Tyrosine Serotonin : Tryptophan

87. The activities of baroreceptors present in the carotid sinus are carried by the afferent fibers of neurons located in

(a) nodose ganglion

(b) geniculate ganglion

(c) petrosal ganglion

(d) spiral ganglion

Ans. (c)

Sol. The petrosal ganglion is a sensory ganglion of the glossopharyngeal nerve. The activities of baroreceptors present in the carotid sinus are carried by the afferent fibers of neurons located in the petrosal ganglion.

88. Which one is NOT a true response of pulmonary J-receptor stimulation by hyperventilation of lung?

(a) Bronchodilation

(b) Decreased heart rate

(c) Apnoea followed by rapid breathing

(d) Low blood pressure

Ans. (a)

Sol. Pulmonary J-receptors are located in the interstitial tissues of the lungs, near the pulmonary capillaries. They are stimulated by a decrease in lung volume, such as that which occurs during hyperventilation.

89. Absorbed monosaccharides in intestinal epithelial cells exit via which one of the following transporters?

(a) GLUT2

(b) GLUT3

(c) GLUT4

(d) GLUT5

Ans. (a)

Sol. GLUT2 is a glucose transporter protein that is primarily responsible for the transport of glucose. It is located on the basolateral membrane of the intestinal epithelial cells. It allows the glucose molecules to leave the cell by facilitated diffusion into the extracellular fluid on the other side of the epithelium

90. A number of statements have been made regarding heme, a component of hemoglobin, as given below:

A. It is synthesized in mature erythrocytes

B. It is synthesized by the condensation of succinyl-CoA and glycine

C. It is synthesized by the condensation of acetyl-CoA and glycine

D. Its synthesis is catalyzed by ä amino levulinate synthase

Which one of the following combinations has both incorrect statements?

(a) A and D

(b) A and C

(c) B and C

(d) B and D

Ans. (b)

Sol. Heme is not synthesized in mature erythrocytes (red blood cells). Instead, heme synthesis occurs primarily in precursor cells of the red blood cells, specifically in the bone marrow, and in certain other tissues.

Heme is not synthesized by the condensation of acetyl-CoA and glycine. Instead, heme biosynthesis starts with the condensation of succinyl-CoA and glycine to form delta-aminolevulinic acid (ALA).

91. Given below are some statements about pituitary hormones:

A. Oxytocin and vasopressin are synthesized in posterior pituitary

B. Prolactin is synthesized from anterior pituitary

C. and MSH are secreted from intermediate lobe of pituitary in adult humans

D. Growth hormone secretion from anterior pituitary is stimulated by hyperglycemia

E. Prolactin secretion is markedly increased by sleep

Choose the incorrect combination of statements from below:

(a) A, C and D

(b) A, B and D

(c) C, D and E

(d) B, C and E

Ans. (a)

Sol. A. Oxytocin and vasopressin are synthesized in posterior pituitary

C. and MSH are secreted from intermediate lobe of pituitary in adult humans

D. Growth hormone secretion from anterior pituitary is stimulated by hyperglycemia

92. Following are the statements made about major functions of some of the neuroglia in normal condition:

A. Oligodendrocytes help maintain K+ level, and contribute to the blood–brain barrier.

B. Microglia are capable of movement and phagocytosis of pathogens and damaged tissue.

C. Astrocytes produce the myelin sheath to electrically insulate neurons of the CNS.

D. Ependymal cells which are ciliated are involved in circulation of cerebrospinal fluid.

Which of the following options represents the combination of all INCORRECT statements?

(a) A and B

(b) A and C

(c) A and D

(d) C and D

Ans. (b)

Sol. Oligodendrocytes are responsible for forming the myelin sheath around axons in the central nervous system (CNS), which electrically insulates neurons, but they do not help maintain K+ (potassium) levels. Astrocytes are primarily involved in regulating K+ levels in the CNS. Astrocytes do not produce the myelin sheath. Oligodendrocytes in the CNS and Schwann cells in the peripheral nervous system (PNS) are responsible for producing the myelin sheath and electrically insulating neurons.

93. The autoregulation of blood flow in the active tissues is partly achieved locally by metabolites accumulated in these tissues. The contributions of different metabolites in this autoregulation are suggested in the following statements:

A. The accumulation of K+ locally in active tissues has vasoconstrictor activity.

B. The increase in osmolality in active tissues causes vasoconstriction.

C. The accumulation of lactate in active tissues may contribute to vasoconstriction.

D. The hypoxia-inducible factor-1 (HIF-1) produced due to local fall in O2 tension in active tissues, initiates the production of different vasodilatory substances.

E. Histamine released from the damaged cells of active tissues increases capillary permeability.

Choose the option with both correct statements.

(a) A and B

(b) B and C

(c) C and D

(d) D and E

Ans. (d)

Sol. D. The hypoxia-inducible factor-1 α (HIF-1) produced due to local fall in O2 tension in active tissues, initiates the production of different vasodilatory substances.

E. Histamine released from the damaged cells of active tissues increases capillary permeability.

94. A human subject can voluntarily inhibit respiration for some time but the subject feels irresistible urge to resume breathing after a while at a point which iscalled the breaking point. The characteristic features of breaking point are suggested in the following statements:

A. The breaking point is shorter in subjects after removal of carotid bodies compared to when they have intact carotid bodies.

B. The breaking point is prolonged if the subject breathes 100% oxygen before breath holding.

C. When the subject hyperventilates with room air before breath holding, the breaking point is delayed compared to when the subject breathes normally before breath holding.

D. he breaking point can be reduced in a subject by making respiratory movements behind a closed glottis.

E. The breaking point is shorter when the subject is told during breath holding that her/his performance is very good compared to a situation when she/he is not told so.

Choose both the correct statements from the following options:

(a) A and B

(b) B and C

(c) C and D

(d) D and E

Ans. (b)

Sol. Breathing 100% oxygen before breath holding can increase the oxygen levels in the body, reducing the urgency to resume breathing. As a result, the breaking point, the moment when the subject feels an irresistible urge to breathe, is prolonged.

Hyperventilation before breath holding reduces the level of carbon dioxide (CO2) in the body. Since the urge to breathe is mainly triggered by the buildup of CO2, a lower level of CO2 delays the breaking point, making the subject feel less urgency to resume breathing.

95. The intake of nutrients is under intricate control. A number of statements are made about factors controlling food intake:

A. Cholecystokinin produced from small intestine stimulates food intake

B. Leptin produced in adipose tissues stimulates food intake

C. Leptin receptors are located in hypothalamus

D. Ghrelin produced in the stomach inhibits food intake

E. Leptin also stimulates the metabolic rate

F. Ghrelin increases secretion of Neuropeptide Y

Choose the combination of all correct statements from the following options:

(a) A, B and C

(b) A, B and D

(c) D, E and F

(d) C, E and F

Ans. (d)

Sol. C. Leptin receptors are located in the hypothalamus.

E. Leptin also stimulates the metabolic rate.

F. Ghrelin increases secretion of Neuropeptide Y.

96. Different waves of EEG (Column A) are listed with their frequencies (Column B) below :

Choose the option with all correct matches of the wave with its frequency.

(a) A-i, B-ii, C-iii, D-iv

(b) A-ii, B-iii, C-iv, D-i

(c) A-iii, B-iv, C-i, D-ii

(d) A-iv, B-i, C-ii, D-iii

Ans. (c)

Sol. Alpha waves (8 - 13 Hz) are often associated with relaxed wakefulness and a calm mental state.

Beta waves (13 - 30 Hz) are generally linked to active and alert mental states, such as during focused attention and problem-solving.

Theta waves (4 - 7 Hz) are related to drowsiness, daydreaming, and some stages of sleep. Delta waves (less than 4 Hz) are typically observed during deep sleep.

97. Which one of the following conditions represents autopolyploidy?

(a) More than two sets of chromosomes, both of which are from the same parental species.

(b) More than two sets of chromosomes, both of which are from the different parental species.

(c) More than two sets of chromosomes only from a single parent.

(d) Duplication of a chromosomal locus leading to spontaneous increase in the copy number of a gene.

Ans. (a)

Sol. Polyploidy is of two types: autopolyploidy and allopolyploidy. Autopolyploidy is the polyploidy condition resulting from the multiplication of the same genome. Allopolyploidy is a polyploid condition formed by crossing different species and doubling the chromosomes of the hybrid.

98. Given below is a pedigree indicating a pattern of inheritance :

Which one of the following options correctly describes the pattern of inheritance shown in the above pedigree?

(a) X-linked recessive

(b) Autosomal recessive

(c) X-linked dominant

(d) Autosomal dominant

Ans. (b)

Sol. Autosomal recessive traits

Trait often skips generations. An almost equal number of affected males and females. If both parents are affected, all children should be affected.

99. In a conjugation experiment between bacterial Hfr strain 'X' and F cell, lac gene enters the recipient in 4 minutes, but the F cells remain auxtotrophic for Leu, Trp, Ura, Glu, Phe and Gly. The mating is then allowed to processed for 20 minutes and lac+ conjugants are selected. Of the lac+ cells,

35% are leu+

98% of trp+

10% of ura+

65% are glu+

0% are phe+

81% are gly+

Select the correct order of the genes as they enter, from the choice given below :

trp+, gly+, glu+, leu+, ura+, phe+, lac+

(a) lac+, phe+, ura+, leu+, glu+, gly+, trp+

(b) phe+, ura+, leu+, glu+, gly+, trp+, lac+

(c) lac+, trp+, gly+, glu+, leu+, ura+, phe+

(d) lac+, trp+, gly+, glu+, leu+, ura+, phe+

Ans. (d)

Sol. Conjugation between different Hfr and F" strains can be used to map the relative positions of genes in the bacterial chromosome. The mapping procedure is based on the fact that genes closer to the origin of transfer will be transferred at a higher frequency than genes farther away from the origin of transfer.

100. The additive nature of a genetic map as suggested by Alfred Sturtevant and T. H. Morgan is possible if there is:

(a) no interference in crossovers.

(b) complete interference in crossovers.

(c) partial interference in crossovers.

(d) variable interference in crossovers dependent on the genetic distances.

Ans. (b)

Sol. Interference refers to the phenomenon where a crossover event in one region of a chromosome affects the likelihood of a crossover occurring in nearby regions. If there is no interference in crossovers, it means that the occurrence of one crossover event does not affect the likelihood of another crossover event happening nearby. If there is complete interference in crossovers, it means that the occurrence of one crossover event prevents any additional crossovers from occurring in nearby regions. The additive nature of a genetic map is possible if there is complete interference in crossovers.

101. Given below is a pedigree indicating a pattern of inheritance:

The following statements are drawn from the above pedigree towards understanding the pattern of inheritance:

A. An affected male does not appear to pass the trait to his sons

B. An affected male appears to pass the allele to a daughter who is unaffected

C. All affected individuals have at least one affected parent

D. The given trait appears to be a recessive one

E. The given trait appears to be an autosomal recessive one

Select the option from the following that has all correct statements:

(a) C and E only

(b) A, B and D only

(c) E only

(d) A, B, C, D and E

Ans. (b)

Sol. In the pedigree, affected males do not have any affected sons, suggesting that the trait is not passed from an affected father to his sons.

In the pedigree, an affected male (individual II-3) passes the trait to his daughter (individual III-1), who is unaffected. This pattern is consistent with the trait being inherited in an autosomal recessive manner.

This pattern is consistent with the trait being inherited as a recessive trait, where individuals need to inherit two copies of the mutated allele (one from each parent) to express the trait.

102. An individual is heterozygous for a reciprocal translocation as shown below in the given diagram:

The following statements are made about segregation of such chromosomes during meiosis and gamete formation:

A. The complexly paired 4 chromosomes fail to segregate, pass into one cell at anaphase I and the cell eventually dies.

B. Chromosomes pair between regions of maximum homology keeping the translocated part unpaired and a normal meiosis occurs.

C. One of the ways the chromosomes segregate is by alternate segregation (N1, N2 moving to one pole and T1, T2 moving to the other pole).

D. Alternate segregation produces non-viable gametes.

E. Reciprocal translocations are considered as crossover suppressors as no gametes with crossover product are produced.

Select the option from the following that describes the meiotic consequences of such translocation correctly:

(a) A only

(b) C only

(c) B and D

(d) A and E

Ans. (b)

Sol. In a reciprocal translocation, two non-homologous chromosomes exchange segments, leading to a rearrangement of genetic material. During meiosis, when homologous chromosomes pair up, the translocated part of the chromosomes remains unpaired. The segregation of such chromosomes can occur in two ways: alternate segregation and adjacent-1 segregation.

103. Two new chemical compounds X and Y are synthesized in a laboratory and tested for their potency as a mutagen. The nature of the mutation produced by these compounds is tested for reversal by other known mutagens and the following results were obtained:

Which statement best describes the nature of the two mutagens?

(a) Compound X produces single base substitutions that generate CG to TA transitions and Compound Y produces insertions or deletions

(b) Compound X produces insertions or deletions and Compound Y produces single base substitutions that generate CG to TA transitions.

(c) Compound X produces single base substitutions that generate CG to TA transitions and Compound Y produces TA to GC transitions.

(d) Compound X produces single base substitutions that generate insertions and Compound Y produces deletions.

Ans. (a)

Sol. The fact that the mutations produced by these compounds can be reversed by other known mutagens suggests that the mutagenic effects of compounds X and Y might not be permanent or irreversible.

104. Which one does not occur as a physiological adjustment during heat acclimatization?

(a) Lowered threshold for start of sweating

(b) Effective distribution of cardiac output

(c) Improved skin blood flow

(d) Increased salt concentration of sweat

Ans. (d)

Sol. Increased salt concentration in sweat, a condition known as hyperhidrosis or "salty sweat," can be caused by various factors. Here are a few possible causes: Cystic Fibrosis: Cystic fibrosis is a genetic disorder that affects the production and function of sweat, mucus, and digestive juices. People with cystic fibrosis have abnormally high levels of salt in their sweat, which leads to salty-tasting skin and increased salt concentration in their sweat.

Medications: Certain medications can affect sweat production and composition, leading to increased salt concentration. For example, some medications used to treat high blood pressure, such as beta-blockers and calcium channel blockers, may alter sweat gland activity and result in saltier sweat.

105. Which one of the following forest type occupies the largest area in India?

(a) Tropical rain forest

(b) Tropical dry deciduous forest

(c) Temperate deciduous forest

(d) Temperate evergreen forest

Ans. (b)

Sol. A tropical dry deciduous forest is a type of forest ecosystem found in regions with a distinct dry season and a wet season. It is characterized by the shedding of leaves by most trees during the dry season as a response to water scarcity.

106. Which one of the following biome is known to occur in India?

(a) Tundra

(b) Boreal forest

(c) Taiga

(d) Alpine grasslands

Ans. (d)

Sol. India does not have tundra, boreal forests, or taiga biomes. These biomes are typically found in high- latitude regions with cold climates. However, India does have alpine grasslands, which are found in the high mountain regions of the Himalayas.

107. The biological species concept defines species as a group of populations that are reproductively isolated from others. However, this definition is not applicable to groups where sexual reproduction has not been observed yet or is extremely rare. Choose the correct option of organisms where biological species concept may therefore not apply:

(a) Monocots and basal angiosperms

(b) Ascomycetes and oligochaetes

(c) Mosses and liverworts

(d) Cyanobacteria and Euglenophyta

Ans. (d)

Sol. The biological species concept, defines species in terms of interbreeding. According to Ernst Mayr, species are groups of interbreeding natural populations that are reproductively isolated from other such groups. Reproductively isolated means that members of the species do not interbreed with members of other species. Biological species concept is not applicable in bacteria due to the lack of sexual reproduction.

108. Which of the following is typically true of invasive species?

(a) They are r-selected

(b) They are K-selected

(c) They are habitat specialists

(d) They are always introduced by humans

Ans. (a)

Sol. Invasive species (also called introduced, exotic, non-native) can be any kind of living organism that is not native to an ecosystem and which has a tendency to change the structure and function of the ecosystem. They can be introduced to an area intentionally or accidentally, and they can quickly become established and spread. A typical invasive species has features similar to r-selected species such as: Rapid reproduction and growth, High dispersal ability, Phenotypic plasticity, Ability to survive in a wide range of environmental conditions.

109. Which of the following correctly represents the relationship between the rate of population growth and population size?

(a)

(b)

(c)

(d)

Ans. (b)

Sol. The relationship between the rate of population growth, dN/dt and population size, N, takes the form of a parabola, reaching a maximum value at a population size of N = K/2. The rate of population growth (dN/dt) is at its hi^hpct when N = K/2 (called the inflection point) and then decreases as it approaches the carrying capacity (K).

110. The Biodiversity Management Committees (BMCs) envisaged under the Biological Diversity Act (2002) and Rules (2004) are constituted at which one of the following administrative levels?

(a) Village

(b) Tehsil / Taluka

(c) District

(d) State

Ans. (a)

Sol. According to the provisions of the Biological Diversity Act and Rules in India, every local body at the village level, known as the Gram Panchayat, is required to constitute a Biodiversity Management Committee (BMC). The BMCs are responsible for promoting conservation, sustainable use, and documentation of biodiversity within their respective jurisdictions, which typically correspond to a village or group of villages. They play a crucial role in the decentralized governance of biodiversity and local participation in biodiversity management and conservation efforts.

111. There is a species that is critically endangered, found in the Russian Far East. It is solitary, but it has been reported that some males stay with females after mating, and may even help with rearing the young. Identify this species.

(a) Amur leopard

(b) Snow leopard

(c) Arctic fox

(d) Black-footed ferret

Ans. (a)

Sol. A species is critically endangered when facing an extremely high risk of extinction in the wild in the immediate future. The Amur leopard is a leopard subspecies native to the Primorye region of southeastern Russia and northern China.

112. Consider a predator species foraging for prey in a habitat, where there are two prey species A and B. Assume the foraging predator can choose from a highvalue prey A and low-value prey B. A and B occur at different frequencies in the environment, so it may take different average times to find the next A or B individual.

Choose the correct option based on the optimal foraging theory.

(a) If it takes too long to search for A, predators may switch to eating B only.

(b) If it takes too long to search for A, predators may eat both A and B, whichever is encountered.

(c) Predators will only feed on B, regardless of search time.

(d) Predators will never feed on B, irrespective of its relative frequency.

Ans. (b)

Sol. Animals search (forage), sense, detect and consume foods. For animals, foraging involves a balance between a food's energy content and the cost of obtaining it. Foraging behavior can directly influence energy intake and individual fitness. In above scenario, if it takes too long to search for the high-value prey species A, predators would not exclusively focus on searching for A but would also consume the low-value prey species B when encountered. This behavior allows them to maximize their energy intake by exploiting the available resources efficiently.

113. Species richness can be measured with the:

(a) abundance of species in an area.

(b) number and the abundance of species in an area.

(c) number of species in an area.

(d) density of species in an area.

Ans. (c)

Sol. Number of species in a community (i.e., richness).

Relative abundance of each species (i.e., evenness).

114. Which one of the following statements in relation to insect wings is not true?

(a) Insect wings are extensions of cuticle and not true appendages.

(b) In beetles, the hind wings function in flight.

(c) Males of many cricket species have forewings modified to bear sound-producing structures.

(d) Flies have a structure called frenulum, which joins the forewing to the hind wing.

Ans. (d)

Sol. Flies do not have a structure called frenulum. The frenulum is a specialized bristle-like structure found in some other insects, particularly in butterflies and moths. It is a small hook-like projection on the hindwing that attaches to a retinaculum (a small catch) on the forewing, helping to keep the wings aligned during flight.

115. Following are a few statements about India's biodiversity:

A. India has 2.4% of the world's land area, but accounts for 12% of all recorded species.

B. India has over 45,000 species of animals and 91,000 species of plants.

C. Four of the globally identified biodiversity hotspots can be found in India.

D. India is estimated to harbour around 60% of the global tiger population.

Which one of the following options represents all correct statements?

(a) Only A

(b) Only C

(c) Both A and B

(d) Both C and D

Ans. (d)

Sol. India is home to several biodiversity hotspots, which are regions with exceptionally high levels of species diversity and are also under significant threat of habitat loss and degradation

India is one of the key countries for tiger conservation and is estimated to be home to around 60% of the world's wild tiger population.

116. A researcher examined the features of newly hatched birds. Species A showed open eyes, down feathers and was able to move around. Species B lacked down feathers and was incapable of walking and its eyes were closed. Given this, choose the correct option.

(a) Species A is altricial and species B is precocial.

(b) Species A is precocial and species B is altricial.

(c) Species A and B are both precocial.

(d) Species A and B are both altricial

Ans. (b)

Sol. Species A is precocial, which means that the offspring are relatively more developed at hatching and are capable of independent movement and behavior shortly after birth. They have open eyes, down feathers, and are able to move around on their own.

Species B is altricial, which means that the offspring are relatively less developed at hatching and are dependent on parental care and protection. They have closed eyes, lack down feathers, and are incapable of walking on their own at hatching. They require care and feeding from their parents until they become more developed and able to fend for themselves.

117. The graph below depicts trajectories (A to D) of some of the major drivers of global environmental change (i to iv) that are mentioned alongside.

Match the trajectories with the correct drivers:

(a) A-iv, B-iii, C-ii, D-i

(b) A-i, B-ii, C-iii, D-iv

(c) A-ii, B-iv, C-i, D-iii

(d) A-iii, B-i, C-iv, D-i

Ans. (a)

Sol. A. The trajectory A represents the increase in N (nitrogen) fertilizer use over time.

B. The trajectory B represents the rise in atmospheric CO2 concentration over time.

C. The trajectory C represents the growth of the human population over time.

D. The trajectory D represents the changes in land area over time, which could include deforestation, urbanization, and other land-use changes.

118. The diagram below depicts the generalized distributional curves (A to D) of allochthonous organic matter and autocthonous production by different autotrophic groups, as a stream transitions to a river.

The following are sources of organic matter:

i. Allochthonous

ii. Autochthonous from phytoplankton

iii. Autochthonous from bottom attached algae

iv. Autochthonous from aquatic macrophytes

Choose the correct option that matches the distributional curves (A to D) to the sources (i to iv):

(a) A-i, B-ii, C-iv, D-iii

(b) A-ii, B-i, C-iii, D-iv

(c) A-iii, B-ii, C-i, D-iv

(d) A-i, B-iv, C-ii, D-iii

Ans. (a)

Sol. A = i. Allochthonous Curve A represents the distribution of allochthonous organic matter, which refers to organic matter derived from external sources and inputs into the stream or river, such as leaves, woody debris, and other plant materials that fall into the water from the surrounding terrestrial environment.

B = ii. Autochthonous from phytoplankton Curve B represents the distribution of autochthonous organic matter derived from phytoplankton, which are tiny free-floating algae that produce organic matter through photosynthesis in the water column.

C = iv. Autochthonous from aquatic macrophytes Curve C represents the distribution of autochthonous organic matter derived from aquatic macrophytes, which are large plants that are rooted in the stream or river bed. They produce organic matter through photosynthesis.

D = iii. Autochthonous from bottom-attached algae Curve D represents the distribution of autochthonous organic matter derived from bottom-attached algae, also known as benthic algae. These algae are attached to rocks and other substrates at the bottom of the stream or river and produce organic matter through photosynthesis.

119. Following are a set of statements about various models of succession:

A. In inhibition model, strong competitive interaction is present as no species is completely superior.

B. In tolerance model, later successional species are neither inhibited nor aided by species of previous stages.

C. In inhibition model, competitive interaction is weak as no species is completely superior.

D. In facilitation model, later successional species are neither inhibited nor aided by species of previous stages.

Which one of the following options represent correct statements?

(a) A and D

(b) B and C

(c) A and B

(d) C and D

Ans. (c)

Sol. A. In inhibition model, strong competitive interaction is present as no species is completely superior.

B. In tolerance model, later successional species are neither inhibited nor aided by species of previous stages.

120. The graphs (A-C) below depict the seasonal variation in plankton biomass in three oceanic regions of Northern hemisphere (i to iii):

Oceanic regions of the world:

i. Tropical oceans

ii. Polar oceans

iii. Temperate oceans

Match the graphs (A to C) to the correct oceanic region (i to iii).

(a) A-i, B-ii, C-iii

(b) A-ii, B-i, C-iii

(c) A-i, B-iii, C-ii

(d) A-iii, B-ii, C-i

Ans. (a)

Sol. i. Tropical oceans: Tropical oceans typically have relatively stable and high plankton biomass throughout the year due to consistent sunlight and warm temperatures.

ii. Polar oceans: Polar oceans experience significant seasonal variation in plankton biomass. During the polar summers, when there is continuous sunlight, plankton biomass increases.

In contrast, during the dark polar winters, plankton biomass decreases.

iii. Temperate oceans: Temperate oceans experience seasonal fluctuations in plankton biomass. Plankton biomass tends to peak in the spring and summer months when sunlight and nutrient availability are abundant, and it declines during the fall and winter months.

121. Which one of the following statements is not correct?

(a) Niche breadth tends to increase with interspecific competition while intraspecific competition tends to decrease it.

(b) Species in unstable environments with fluctuating resource availabilities tend to have broad niche breadths.

(c) K-strategists are likely to be better competitors than r-strategists in a climax community.

(d) Diffuse competition increases with niche dimensionality.

Ans. (a)

Sol. Niche breadth tends to decrease with interspecific competition: When multiple species compete for the same resources (interspecific competition), they may specialize and reduce their niche breadth to avoid direct competition, leading to niche partitioning.

122. The following table shows a list of organisms and associated adaptive characteristics.

Select the correct option that matches the name of the organism with their correct adaptation.

(a) A-ii, B-i, C-iv, D-iii

(b) A-iii, B-ii, C-i, D-iv

(c) A-ii, B-iii, C-iv, D-i

(d) A-iv, B-i, C-ii, D-iii

Ans. (a)

Sol. A. Coral snake - Mimicry

B. Crystal Jelly - Bioluminescence

C. African lungfish - Aestivation

D. Monarch butterflies - Aposematism

123. The correct hierarchy of geological times is:

(a) eon > era > period > epoch

(b) period > era > epoch

(c) epoch > period > era > eon

(d) era > eon > period

Ans. (a)

Sol. eon > era > period > epoch

This is the correct order of the hierarchical divisions of geological time. An eon is the largest division of time, followed by era, period, and epoch. Each division represents a different scale of time and encompasses various geological and biological events and processes

124. Any movie that features dinosaurs should also have which of the following combinations of geological age-appropriate organisms? Choose the correct combination.

(a) Humans, angiosperms and gymnosperms, birds

(b) Early diverging angiosperms, reptiles, amphibians

(c) Apes, gymnosperms, birds

(d) Early diverging gymnosperms, amphibians, reptiles

Ans. (b)

Sol. Movies featuring dinosaurs should include organisms that existed during the same time period. Earlydiverginggy mnosperms, amphibians, reptiles were present during the same time as dinosaurs

125. Which one of the following traits would hypothetically NOT be considered for preferential selection during domestication of the corresponding crops listed below?

(a) Increased fruit size of tomato

(b) Reduced spininess in okra

(c) Shattering seeds of corn

(d) Increased oil content of mustard

Ans. (c)

Sol. Shattering seeds are seeds that disperse naturally when they are ripe. However, it is not a desirable trait in domesticated plants, as it makes it difficult to collect the seeds. Therefore, farmers would not preferentially select for shattering seeds in corn. Instead, they would select for seeds that do not shatter, so that they can be easily collected and stored.

126. The following statements were made about adaptive radiation:

A. Adaptive radiation is a kind of divergent evolution driven by ecological diversification.

B. Adaptive radiation is the divergence of unrelated taxa into different niches.

C. Adaptive radiation is rare on archipelagos removed from the mainland.

D. Processes unrelated to niche exploitation can be major drivers of species diversification

Choose the option that represents all correct statements.

(a) A and B

(b) C and D

(c) B and C

(d) A and D

Ans. (d)

Sol. A. Adaptive radiation is a kind of divergent evolution driven by ecological diversification. This statement is correct. Adaptive radiation is a form of divergent evolution in which a single ancestral species gives rise to a variety of species, each adapted to different ecological niches.

D. Processes unrelated to niche exploitation can be major drivers of species diversification.

This statement is also correct. While ecological diversification and niche exploitation are essential factors driving adaptive radiation, other processes unrelated to niche exploitation, such as geographic isolation and genetic factors, can also play a significant role in species diversification during adaptive radiation.

127. Which one of the following statements best describes Bateman's principle?

(a) Female gametes (eggs) are costlier than male gametes (sperms).

(b) Reproductive variance is greater in males than in females.

(c) Females are more likely to provide parental care than males.

(d) Males use costly displays to advertise their genetic quality.

Ans. (b)

Sol. This principle, proposed by Angus John Bateman, suggests that in many species, males typically have a higher potential reproductive success due to their ability to produce many offspring through multiple mates. In contrast, females often have a limited number of offspring due to the costs associated with reproduction, such as investing time and resources in gestation and nurturing. This leads to greater variability in male reproductive success compared to females.

128. The diagrams A-D below shows the relative abundance of major groups of plants (refer to legend) in four different geological periods (Devonian, Carboniferous, Tertiary and Cretaceous).

Match the diagrams (A to D) with the correct geological period.

(a) A-Tertiary, B- Carboniferous, C-Devonian, D-Cretaceous

(b) A-Cretaceous, B- Devonian, C-Tertiary, D-Carboniferous

(c) A-Tertiary, B- Cretaceous, C-Carboniferous, D-Devonian

(d) A-Devonian, B- Tertiary, C-Cretaceous, D-Carboniferous

Ans. (b)

Sol. A- cretaceous, B- Devonian, c- tertiary, d-carboniferous.

129. Individuals belonging to the fossil genera Calamites are considered to be upright arborescent plants. They were characterized by stems which mostly arose from subterranean rhizomes. The cross sections of young stems showed the presence of a central pith canal and collateral vascular bundles with carinal canals. To which of the following extant genera is this plant most similar?

(a) Psilotum

(b) Selaginella

(c) Equisetum

(d) Rhynia

Ans. (c)

Sol. Equisetum, commonly known as horsetails, is a modern-day genus of vascular plants that shares several similarities with Calamites. Equisetum also exhibits upright, hollow stems with central pith canals and collateral vascular bundles with carinal canals. The resemblance between Calamites and Equisetum is so close that they are often referred to as living fossils due to their similar morphological features and ancient lineage.

130. The following table shows a list of evolutionary processes and their associated characteristics :

Select the correct option that matches the evolutionary process to its silent characteristic.

(a) A-iv, B-i, C-ii, D-iii

(b) A-i, B-iv, C-iii, D-ii

(c) A-iv, B-iii, C-i, D-ii

(d) A-iv, B-ii, C-iii, D-i

Ans. (b)

Sol. A. Parallelism - i. Closely related groups evolve similar characteristics. This is correct. Parallelism refers to the evolutionary process where closely related species or groups evolve similar characteristics independently due to shared ancestry or similar environmental pressures.

B. Convergence - iv. Two or more distantly related groups acquire similar characteristics. This is correct. Convergence is the evolutionary process where distantly related species or groups independently evolve similar characteristics in response to similar selective pressures or ecological niches.

C. Introgression - iii. Crossbreeding between species is mediated by repeated backcrossing. This is correct. Introgression is the process of genetic exchange between different species through repeated backcrossing, leading to the transfer of genetic material from one species to another.

D. Hybridization - ii. Individuals of different species crossbreed. This is correct. Hybridization refers to the process of individuals from different species mating and producing offspring, which are hybrids with genetic material from both parent species.

131. The following tables lists major food crops and the region of domestication :

Which one of the following options represents the correct match between the food chap and its region of domestication?

(a) A-ii, B-iii, C-i, D-iv

(b) A-iii, B-i, C-ii, D-iv

(c) A-iv, B-i, C-iii, D-ii

(d) A-ii, B-iv, C-iii, D-i

Ans. (b)

Sol. A. Banana - Middle East

B. Mung bean - India

C. Sorghum - Africa

D. Wheat - Middle East

132. The following are a set of characteristics found in the animal kingdom:

A. The body is usually streamlined. Some have spindle-shaped or elongated body.

B. The body is covered with thick-seated scales, which provides protection to the internal organs.

C. They may be herbivores or carnivores, oviparous or ovoviviparous.

D. The nervous system comprises of the brain and ten pairs of the cranial nerves.

E. All of them are oviparous and exhibit sexual dimorphism.

Select the correct set of characterizing features for the Class Pisces.

(a) B, D and E only

(b) A, B and D only

(c) B, C, D and E

(d) A, B, C and D

Ans. (d)

Sol. A. The body is usually streamlined. Some have spindle-shaped or elongated body.

B. The body is covered with thick-seated scales, which provides protection to the internal organs.

C. They may be herbivores or carnivores, oviparous or ovoviviparous.

D. The nervous system comprises of the brain and ten pairs of the cranial nerves.

133. Which one of the options correctly represents organisms from the subphyla Chelicerata, Myriapoda, and Hexapoda, in this specific sequence?

(a) Arachnids, horseshoe crabs, centipedes

(b) Horseshoe crabs, centipedes, springtails

(c) Lobsters, millipedes, silverfish

(d) Arachnids, insects, crabs

Ans. (b)

Sol. Horseshoe crabs (Chelicerata), centipedes (Myriapoda), springtails (Hexapoda).

134. The following picture represents a gel profile of a pair of DNA markers observed in parents P1 and P2, their F1 progeny and F2 progeny. Four different profiles were observed in case of F2. The number of F2 progeny showing a given profile is indicated in brackets

Based on the above observation, which one of the following statement is correct?

1. Co-dominant DNA markers were used for this study.

2. The polymorphic DNA bands represents two independent genes.

3. If the P1 parent was crossed to the F1 individual, the progeny will show all the four profiles as observe in the case of F2 progeny.

4. If an F2 progeny which does not show either of the DNA markers (last one of the above gel) is crossed to a P1 individual, the obtained progeny will have two types of individual, one which shows a band and the other where no band is observed.

(a) Co-dominant DNA markers were used for this study.

(b) The polymorphic DNA bands represents two independent genes.

(c) If the P1 parent was crossed to the F1 individual, the progeny will show all the four profiles as observed in the case of F2 progeny.

(d) If an F2 progeny which does not show either of the DNA markers (last lane of the above gel) is crossed to a P1 individual, the obtained progeny will have two types of individual, one which shows a band and the other where no band is observed.

Ans. (b)

Sol. The F2 progeny exhibiting a Mendelian phenotypic ratio of 9:3:3:1 suggests the independent assortment of two genes and complete dominance in their allelic relationship. This means that each gene is inherited independently of the other, and the dominant allele will always be expressed over the recessive allele.

135. The measurement of distance based on counting steps or number of vertical bars by insects for navigation is called:

(a) path integration.

(b) allocentric coding.

(c) odometry.

(d) alignment image-matching.

Ans. (c)

Sol. Odometry refers to the process by which an organism estimates its position or distance traveled based on internal sensory information. Insects, such as ants, bees, and desert ants, use odometry as a navigational strategy to keep track of their movement. They count their steps or use other internal mechanisms to estimate the distance they have traveled, allowing them to navigate back to their nest or a desired location.

136. What is the 50th percentile of the numbers 9, 5, 11, 3 and 2?

(a) 5

(b) 6

(c) 9

(d) 15

Ans. (a)

Sol. 50th percentile is known as the median and it cuts the data set in half. Half of the given numbers lie below the median and half lie above the median. So, the 50th percentile of the numbers is 5.

137. In remote sensing, which one of the following formulae is used for the calculation of normalized difference vegetation index (NDVI)?

(a) RED / (NIR + RED)

(b) RED / (NIR – RED)

(c) (NIR + RED) / (NIR – RED)

(d) (NIR – RED) /(NIR + RED)

Ans. (d)

Sol. NDVI = (NIR-RED)/(NIR + RED)

NIR - light reflected in the near-infrared region

RED - light reflected in the red region

138. The radioactive isotope of an element has a half-life of 100 hours. How many hours will it take for of the source amount to decay?

(a) 50

(b) 400

(c) 250

(d) 1000

Ans. (b)

Sol. Given initial amount (N0) = 1, Nt = 15/16, final amount (N) = 1 – 15/16 = 1/16

N/N0 = 1/16 or, N0/N = 16

As we know that,

Taking natural log on both sides, we have

t = 4 × 100 = 400 hours

139. Which one of the following correctly describes the spectroscopic experiment that would help distinguish between a á helix, a 310 helix and a π helix?

(a) Near UV absorption spectrum between 250-300nm

(b) Fluorescence emission spectra between 350-400nm

(c) 1H NMR spectroscopy involving Hydrogen/Deuterium exchange

(d) Near UV Circular Dichroism spectrum between 250-300nm

Ans. (c)

Sol. Answer D (However, answer given by NTA is C)

Circular Dichroism (CD) spectroscopy is a technique commonly used to study the secondary structure of proteins. It measures the differential absorption of left-handed and right-handed circularly polarized light by a molecule. The near UV region (250-300 nm) of the CD spectrum is particularly informative for analyzing protein secondary structures such as 310 and rc-helices.

140. Dixon plot is used to study the enzyme inhibition by plotting various expressions of velocity (v) and inhibitor concentration (I) on the X-axis (Column A) and Y-axis (Column B) as given below :

Which one of the following options is the correct combination from columns A and B to draw the Dixon plot?

(a) Column A -iv, Column B- i

(b) Column A -i, Column B- ii

(c) Column A- ii, Column B- iii

(d) Column A- iii, Column B- iv

Ans. (a)

Sol. A = iv , B = i

141. If the weights of 10,000 seeds from 100 individuals of a tree species are measured, which one of the following distributions is expected?

(a) Binomial

(b) Poisson

(c) Gaussian

(d) No predictable distribution

Ans. (c)

Sol. The Gaussian distribution is a continuous probability distribution that is symmetric around its mean value. It is commonly observed in natural phenomena, including measurements of biological traits like seed weights.

142. Given below are a few statements about plant breeding and transgenesis:

A. Recombinant inbred lines and double haploid populations have high levels of genetic homozygosity.

B. Gene pyramiding involves introducing different genes for resistance to a specific pest in different genotypes of a plant species.

C. Agrobacterium strains with a disarmed Ti plasmid do not require vir genes for transfer of T-DNA.

D. Molecular breeding can be used for crop improvement if the trait of interest is present in naturally occurring populations of the plant.

Which one of the following options represents a combination of incorrect statements?

(a) A and B

(b) A and C

(c) B and D

(d) B and C

Ans. (d)

Sol. B. Gene pyramiding involves introducing different genes for resistance to a specific pest in different genotypes of a plant species.

C. Agrobacterium strains with a disarmed Ti plasmid do not require vir genes for transfer of T-DNA.

143. Given below is a schematic representation of the T-DNA region of a binary vector used for genetic transformation of plants.

LB: Left Border, RB: Right Border, M: Marker gene, P: Passenger gene, pA: poly-adenylation signal, Pr1: Promoter of M gene, Pr2: Promoter of P gene, E: Restriction enzyme (sites) used for digestion of genomic DNA for Southern blotting, Probe 1 and Probe 2: probes used for Southern blotting. Transgenic plants generated using the above construct were subjected to Southern hybridization following digestion of genomic DNA with restriction enzyme 'E', to identify true single copy integration events from LB and RB flanks of the DNA.

Based on the above information, the following statements are made:

A. Single copy events from the LB flank identified using Probe 1 would show two hybridization bands on the Southern blot.

B. Single copy events from the RB flank identified using Probe 2 would show a single hybridization band on the Southern blot.

C. For true single copy events, one hybridization band would be of the same length for each of the two probes used for hybridization.

D. True single copy events would show two bands each for copy number on the left border and right border flanks.

E. There would be no similar hybridization band obtained using Probe 1 and Probe 2.

Which one of the following options represents only correct statements?

(a) A, B and D

(b) C, D and E

(c) B, C and E

(d) A, C and D

Ans. (d)

Sol. When there is a single copy integration event from the LB (Left Border) flank, the Southern blot using Probe 1 would show two hybridization bands, one from the LB region and one from the genomic DNA, indicating a single copy integration

True single copy events will have one hybridization band of the same length for each of the two probes used for hybridization, indicating that there is a single copy integration of the transgene.

True single copy events will show two bands each for the copy number on the LB (Left Border) and RB (Right Border) flanks when subjected to Southern hybridization, confirming the presence of a single copy integration of the transgene on both flanks.

144. Given below are names and recognition sequences of a few restriction enzymes that are used for cloning experiments. The cleavage site ofeach enzyme is indicated by *.

EcoRI – G*AATTC

HindcII – GTY*RAC

EcoRV – GAT*ATC

BamHI – G*GATCC

BgIII – A*GATCT

Given below are different vector (Column A) and insert (Column B) fragments generated by digestion using the above enzymes :

Which one of the following options represents the correct combination of the vector and insert respectively, which generate compatible ends of ligation?

(a) A – ii, B – v, C – iv

(b) B – ii, D – iv, E – i

(c) A – v, C – i, E – iii

(d) C – iv, D – i, E – ii

Ans. (b)

Sol. B - EcoRV (GAT'ATC) = ii D - Bglll (A*GATCT) = iv E - EcoRI (G'AATTC) = i

145. Following statements are made regarding the properties of two- photon microscopy over traditional confocal microscopy:

A. By using longer wavelength, two-photon microscopy induces less photobleaching of the tissue preparation.

B. By using shorter wavelength, two-photon microscopy induces less photobleaching of the tissue preparation.

C. The intensity of fluorescence emitted by the sample will remain the same even if only one of the two exciting photons impinge on the sample.

D. No fluorescence is detected unless two exciting photons simultaneously impinge on the sample.

Which one of the following combination of statements is correct?

(a) A and C

(b) A and D

(c) B and C

(d) B and D

Ans. (b)

Sol. By using longer wavelength, two-photon microscopy induces less photobleaching of the tissue preparation.

No fluorescence is detected unless two exciting photons simultaneously impinge on the sample.